Chapter 24

Practice Test 1: Answers and Explanations

REASONING THROUGH LANGUAGE ARTS

Section 1

1. A

This development question asks which sentence from the passage supports the idea that Ingred’s relatives have had military experience. Since several quotations are related to the major theme of the recent war, consider what is implied as well as what is directly stated. Choice (A) is correct: It talks about “Mr. Saxon, Egbert, and Athelstane” which follows the first sentence in paragraph 1 which states that the family has been reunited; therefore, these are members of the Saxon family. Additionally, the word “demobilized” and the phrase “had hardly yet settled down to civilian life” support the idea that these members of Ingred’s family had served in the war. Choices (B) and (C) are incorrect because, while they do mention that there has been a war, neither choice mentions Ingred’s family. Although (D) indicates that Ingred’s relatives have returned from somewhere, there is no mention of the war. The correct answer is (A).

2. C

This language use item requires test takers to determine which definition of the word “native” matches its use in paragraph 1. Each of the answer options reflects an actual definition of the word “native,” so look closely at the way the word is used in context. The word is used as a part of the sentence “They had joined the rest of the party at Lynstones before returning to their native town of Grovebury.” Here, there is a contrast drawn between where the family is at the time of the narrative—the summer home at Lynstones—and their final destination, which is their family home at Grovebury. Here, “native” means “place of origin.” Choices (A) and (B) are incorrect, because while there are many descriptions of the “natural” and perhaps “wild” beauty of Lynstones, “native” here refers to Grovebury, which is not described as a place in nature. Choice (C) is correct: “original” means “first” or “earliest” which is consistent with the idea that the Saxons are returning to their “place of origin.” Finally, (D) is incorrect, because while the Saxon family home might indeed be “inherited,” there is no evidence to support this in the text. The correct answer is (C).

3. B

This structure question asks about how the sentence describing the landscape enhances the story. The passage uses the phrase “glorious run” to emphasize Ingred’s positive feelings during the trip, and paints a detailed picture of the landscape with phrases like “glimpses of the sea,” “craggy,” and “billowy masses.” Altogether, the sense is of an appreciation of the natural scenery. Choice (A) is incorrect: While the next sentence does mention that “Egbert exceeded the speed-limit,” it goes on to say, “he had the excuse of a clear road before him,” so the passage emphasizes that the trip is picturesque, not dangerous. Choice (B) is correct: Since the contrast between Ingred’s feelings about her summer holidays versus returning to school is a main theme of this excerpt (first described in paragraph 1), the description of nature and use of “glorious” reinforces Ingred’s feelings about her summer vacation home. Choice (C) is incorrect because although the quotation does describe the sea and the cliffs as having different characteristics, the difference between the sea and the cliffs is not a major theme of the story. Even though Egbert appears in this part of the narrative, the quoted description of the landscape does not reveal anything about his character or function in the story, so (D) is incorrect. The correct answer is (B).

4. A

This structure question asks about the significance of the “place” mentioned in the quotation. The sen-tence included in the question says, “But surely the Red Cross cleared out ages ago,” meaning that the “place” had been used by the Red Cross during the war. To find out exactly what the “place” is, look further in paragraph 12. Avis asks Ingred, “I suppose you’re going back to Rotherwood, aren’t you?” Since one of the major themes of the passages is Ingred’s feelings about having to return to her family home as soon as the summer is over, the “going back to Rotherwood” Avis mentions must be that return to the Saxon family home. Therefore, (A) is correct. Choice (B) is incorrect because, while the passage does mention the Red Cross, Avis is specifically referring to a house, not a town, in paragraph 16: “I always envied you that lovely house.” Choice (C) is incorrect: even though the quotation mentions that the “whole place has been done up,” there is no evidence that this was done to a ballroom for the purpose of a dance. Choice (D) is incorrect, because the “place” is Ingred’s home, not her school. The correct answer is (A).

5. A

This character development question asks about Ingred’s character based on her response to Avis. Use the dialogue that ends with Ingred’s response in paragraph 15 to get a sense of the entire exchange between the characters. When Avis asks, in paragraph 12, whether Ingred is going back to Rotherwood, Ingred gives a vague response. When Avis persists, “But surely the Red Cross cleared out ages ago,” then Ingred responds, “Oh, yes!” (paragraph 15) in a voice “a little strained.” Since Ingred’s voice is “strained” when she discusses her return, this is consistent with previous information that Ingred enjoys her holidays and does not want to think about the time beyond summer (paragraph 1) when she will return to her family home. Therefore, (A) is correct: Ingred is reluctant to talk about going back to Rotherwood. Choice (B) is incorrect, because while Ingred is technically “agreeing” with Avis, the strain in her voice shows reluctance. Choice (C) is incorrect, because while “strained” does sometimes mean “exhausted,” in paragraph 15 “strained” means “forced,” showing that Ingred does not really want to think about her eventual return. Choice (D) is incorrect because even though Ingred says “Oh, yes!” emphatically, her inner emotion is not one of enthusiasm. The correct answer is (A).

6. This plot development item asks the reader to arrange the sequence of events in the excerpt in order in a chart. The correct order from first to last:

1. Event (d): The Saxons unite at Lynestones. This occurs early in the narrative in paragraph 1: The first sentence says “the whole family was reunited” and the third sentence says “They had joined the rest of the party at Lynstones.”

2. Event (a): Egbert drives Ingred to the beach. Following the general description of Ingred’s feelings in paragraph 1, a specific day is described in paragraph 2, during which Egbert takes Ingred for a motorcycle ride: “Egbert was anxious to set off, so Hereward took his place on the luggage-carrier, and, after some back-firing, the three started forth.”

3. Event (b): Ingred is invited to tea. After arriving at the seaside, Ingred meets Avis, who invites her to tea: “Come along and have some tea with us,” (paragraph 8).

4. Event (c): Avis talks about a fancy-dress dance. As Avis and Ingred talk, Avis becomes enthusiastic about the idea of dancing: “I do think a fancy-dress dance is about the best fun on earth,” (paragraph 18).

7. This character development item asks the reader to identify three adjectives that accurately describe Ingred. The three correct adjectives:

8. D

This development item requires making a prediction about Avis based on an overall understanding of the details that relate to her. Avis appears in the narrative in paragraphs 3–20, so use the details in these paragraphs to eliminate answers that don’t conform to the passage. Choice (A) is incorrect: Avis is described as “a schoolfellow” (paragraph 3), so she has been to school with Ingred before. Choice (B) is incorrect because Avis doesn’t mention hosting, but talks of Ingred possibly giving parties: “you’ll be giving all sorts of delightful parties, won’t you?” (paragraph 16). Eliminate (C), because it is Ingred who is visiting for the afternoon, not Avis (paragraph 7). Choice (D) is correct, because Avis sighs “only a few days now,” and Ingred tells Avis she is staying “a fortnight more (paragraphs 10–11). Even without knowing the definition of fortnight, Ingred indicates that she is staying “more” than Avis, so the correct answer is (D).

9. This item asks the reader to identify statements that express the EPA’s purposes for writing its article. The two correct statements:

10. C

This question asks which answer is supported by the Department of Energy article, which covers methods of fuel economy. Choice (A) is incorrect, because the article never states that other methods are replaced or “no longer necessary” due to the start-stop system. Choice (B) is incorrect because aftermarket alternative fuel conversions are mentioned only in the other article, not the Department of Energy article. Choice (C) is correct: This article is about methods of fuel economy, including better driving habits (paragraph 6). Choice (D) is incorrect, because this is the main idea of the EPA article, not the Department of Energy article. The correct answer is (C).

11. B

This structure question asks about the EPA’s purpose for including a particular detail, in this case, the sentence, “Very few manufacturers have applied for this program in the past 10 years.” The main idea of this passage is to warn the reader about fuel additives and devices to improve fuel economy, so consider how this detail relates to the main idea. Choice (A) is incorrect: There is no information about how long the program takes. Choice (B) is correct: Paragraph 3 goes on to say, “Without this report, the EPA has no information about the safety of the device or its impact on fuel efficiency.” So the EPA cannot verify the claims of manufacturers who do not apply for the program, and consumers should be wary. Choice (C) is incorrect: This sentence in the question refers to applying for the program, not the testing that occurs later. Choice (D) is also incorrect: The tone and purpose of the EPA article is directed not at the manufacturer but at the consumer. The correct answer is (B).

12. C

This structure question asks about how a figure relates to the overall purpose of the article by the Department of Energy, which covers methods of fuel economy. The chart illustrates the guideline about driving speed in paragraph 6: “Each 5 MPH you drive over 60 MPH can reduce your fuel economy by 7%.” Choice (A) is incorrect, however, because reducing fuel economy 7% by speeding does not equate to improving fuel economy 5% by slowing down. Choice (B) is incorrect because this is a question about the role of the figure in the article, and the chart does not reflect data about vehicle maintenance. Choice (C) is correct: The chart visually illustrates the loss of fuel economy with increased speed. Finally, (D) is incorrect because the chart has no information about driving conditions. The correct answer is (C).

13. B

This is a comparison question about the similarities in the two articles. Both articles address fuel economy, but the first article warns about devices and the second article recommends a device. Choice (A) only reflects the second passage by the Department of Energy, so this answer is incorrect. Choice (B) is correct: The EPA warns about aftermarket devices while the Department of Energy recommends the start-stop system. Again, (C) only reflects the second passage by the Department of Energy, so this answer is incorrect. Choice (D) is wrong because neither passage offers a way to verify advertisers’ claims. The correct answer is (B).

14. A

This structure question asks which specific detail is included in the EPA’s article. Choice (A) is correct: Paragraph 3 warns, “Tampering with your car’s emissions control system is punishable by significant fines,” so there are potential hidden costs when taking these measures. Choice (B) contradicts the passage: The EPA says, “Most devices tested in earlier years had a neutral or negative effect on fuel economy” (paragraph 3). Eliminate (C), which also contradicts the passage: While the EPA registers additives (paragraph 2), it makes no similar claims about devices. Finally, (D) is too strong: Although paragraph 3 states, “Without this report, the EPA has no information about the safety of the device,” it’s possible that the EPA does have reports on some devices so some information is available. The correct answer is (A).

15. B

This question asks the reader to examine the structural relationships in paragraphs 6 and 7 of the Department of Energy’s article. Choice (A) is incorrect: The phrase “doesn’t require you to drive differently” (paragraph 7) refers to driving with a start-stop system, not driving in general, so the parts of the article do not contradict one another. Choice (B) is correct: The first item in the list of recommended habits is to avoid idling (paragraph 6), and paragraph 7 offers a system that virtually eliminates idling. Choice (C) is incorrect because paragraph 7 does not directly compare the start-stop system to other methods that reduce fuel consumption. Finally, (D) is incorrect because the start-stop system is not installed aftermarket, but comes with new vehicles (“as of the 2014 model year, they are available on about one hundred conventional vehicle models“). The correct answer is (B).

16. A

This language use question asks the reader to examine the effect of the use of the word “robust” in the Department of Energy’s article. Paragraph 7 recommends start-stop systems. This section acknowledges “it may take some time for you to get used to” but then immediately follows with “Most systems are robust and easy to use,” to reassure and persuade the reader, since “robust” means “sturdy.” Therefore, (A) is correct. Choice (B) is incorrect, because even though the sentence claims the system is “easy to use,” this is not the meaning of the word “robust.” Choice (C) is incorrect because although another meaning of “robust” is “hefty,” that does not fit the context of the paragraph. Choice (D) is incorrect, because even though paragraph 7 claims the system is “only a few hundred dollars,” this is not the meaning of the word “robust.” The correct answer is (A).

17. The option that correctly completes the sentence for each “Select” option:

Drop-Down Item 1: Sentence fragments and run-ons

Option 1 is correct: Appropriate punctuation (a comma) separates the dependent clause “just as I do” from the independent clause “Surely you enjoy living in such a quiet and peaceful neighborhood.”

Option 2 is incorrect because it incorrectly uses a period, creating a sentence fragment. Option 3 provides punctuation between the independent and dependent clause, creating a run-on sentence. Option 4 treats a somewhat lengthy phrase as though it is an independent clause, separated by a semicolon. However, this phrase is actually a sentence fragment and therefore cannot be separated by a semicolon.

Drop-Down Item 2: Misplaced modifiers or illogical word order

Option 2 is correct: In this sentence, the phrase “you must be aware of” appropriately refers to “the alarming increase in burglary incidents,” while “via news reports and social media reports,” modifies “you must be aware.” The phrases appear in an appropriate order with no dangling modifiers or unclear antecedents.

Option 1 is incorrect: In this order, the phrases appear to indicate that the “burglary incidents” are “via news reports.” Option 3 is incorrect: In this option, the phrase “you must be aware of by now” seems to refer to “social media posts” and creates a sentence fragment. Option 4 is incorrect, because this option presents the construction “social media posts in our neighborhood” so that it appears the social media posts (instead of the burglaries) happen in the neighborhood.

Drop-Down Item 3: Verb tense, parallelism

Option 3 is correct: This option correctly uses the present tense “are concerned” to parallel “propose” and “consider” later in the sentence.

Option 1 is incorrect: The use of “concerning” and “we all do” results in the meaning “we all do concern our community” rather than “we are all concerned about our community.” Option 2 is incorrect, because the past tense “were” does not match the present tense “propose” and “consider” later in the sentence. Option 4 is incorrect because the past tense “were” does not match the present tense “propose” and “consider”.

Drop-Down Item 4: Pronoun usage

Option 2 is correct: The option correctly uses the singular objective pronoun case “it” to match to “check or money order.”

Option 1 is incorrect: The sentence is asking the reader to “send a check or a money order,” only one, not both, so a singular pronoun is required. Option 3 is incorrect: Although “that” can function as a pronoun, the word “that” refers to something specific that is clearly understood, as in “that cat (of several cats).” For “that” to work in the context, a more complete phrase like “that payment” would be required. Option 4 is incorrect for similar reasons: For “one” to work in context, the entire phrase “one of them” would be required.

Section 2: Extended Response

As noted in Chapter 10, the Extended Response essay is evaluated based on three traits. You can earn up to 2 points for each trait for a possible maximum of 6. Here, we’ve provided two sample responses to give you an idea of what a score of 1 and 2 looks like for each trait.

Sample Response A

The argument of whether or not cap-and-trade is beneficial is best supported by the benefits of cap-and-trade. The environment and air quality is very important and so emissions control is needed. Cap-and-trade is something that has been implemented with success and that saves lives. It is a way for the EPA to control emissions by giving businesses and industries permits.

Since nitrous oxide and sulfur dioxide are harmful it makes sense to try to control them to try to keep the air clean. The benefits argument uses studies that have shown that SO2 emissions have dropped 40% since the 1990s, and acid rain levels have dropped 65% since 1976. Also the benefits argument mentions cap-and-trade has reduced emissions because businesses are willing to be part of a low-cost program. These are all good points for the benefits of cap-and-trade. If cap-and-trade was an idea that didn’t work, it wouldn’t have been in use for so long, starting from 1976.

The argument against cap-and-trade states that the results are brought into question because there were other regulations happening at the same time as the Acid Rain Program. They also claim that the permit process causes bad incentives because free permits mean companies might not try very hard to reduce emissions. They claim that if a company reduces emissions, then their cap might be lower next year, so a company might try not to reduce emissions. But there are no statistics to show that this would happen.

If those are the argments that are made then people just need to rely on the actual results. Read the statistics and see how the Acid Rain Program and other programs are actually working. The EPA does a good job of making sure businesses can’t pollute too much.

Trait 1—Creation of Arguments and Use of Evidence

Score: 1

The writer of this response makes an argument in favor of cap-and-trade (“The argument of whether or not cap-and-trade is beneficial is best supported by the benefits of cap-and-trade”) through a somewhat unsophisticated analysis. The response contains some evidence from the source text to support the central position, but this evidence is presented as references that only summarize the source text (“The benefits argument uses studies that have shown SO2 emissions have dropped 40% since the 1990s, and acid rain levels have dropped 65% since 1976”). There is some analysis of the issue and evaluation of the evidence for the arguments, but it is minimal (“If those are the argments that are made then people just need to rely on the actual results” and “These are all good points for the benefits of cap-and-trade”). Overall, the writer makes an argument supported by some analysis and some evidence from the source text. Therefore, this Response A earns a score of 1 for Trait 1.

Trait 2—Development of Ideas and Organizational Structure

Score: 1

The writer establishes an organizational structure in this response by providing a comparison of the two arguments presented. In the introduction, the writer takes a position (“The argument of whether or not cap-and-trade is beneficial is best supported by the benefits of cap-and-trade”). The second paragraph addresses the benefits of cap-and-trade and shows a clear progression of ideas. In the paragraphs, main points are developed in a general sense, but the details are not as well developed. In the second paragraph, a new thought is presented about the benefits of cap-and-trade (“If cap-and-trade was an idea that didn’t work, it wouldn’t have been in use for so long, starting from 1976.”). The third paragraph focuses on the unwelcome possibilities of cap-and-trade, summarizing the source text and offering an assessment (“They claim that if a company reduces emissions, then their cap might be lower next year, so a company might try not to reduce emissions. But there are no statistics to show that this would happen.”). The conclusion is a general explanation of why the second position is better supported. The writer’s word choice is sufficiently competent and the response’s tone is acceptable. Overall, the response is somewhat organized and focused, but the ideas are underdeveloped. Therefore, Response A earns a score of 1 for Trait 2.

Trait 3—Clarity and Command of Standard English Conventions

Score: 1

This brief response shows generally correct sentence structure, but without sentence variety. There is unreliable control of standard English conventions with regard to subject verb agreement (“The environment and air quality is very important”) and punctuation usage. These errors do not interfere with comprehension, however, and this response is an acceptable example of draft writing. Therefore, Response A earns a score of 1 for Trait 3.

Sample Response B

There is much debate about whether cap-and-trade as a strategy for controlling emissions is effective. It is an important debate because air quality is a pressing issue today and addressing harmful emissions can possibly save lives. In this discussion of cap-and-trade, the proponents have made the better argument. They bring up several benefits to cap-and trade, including advantages to business. The pro cap-and-trade article then uses statistics to support its position. The second article from the critics’ standpoint uses mostly hypothetical assertions and warnings of what might happen, with no statistical data to support the position.

In the first article, an outline of the benefits of cap-and-trade are supplied to show what makes the practice was effective—cost, accountability, and the support of federal and state standards. The first argument then provides examples of some specific successful programs: the nationwide Acid Rain Program (ARP) and the regional NOx Budget Trading Program. The first article then cites several statistics pointing to improvements in both SO2 and NOx. According to the article, improvements in emissions mean saved lives, and saving lives is very strong evidence for using cap-and-trade.

The second article then calls the statistical evidence into question by pointing out that, in addition to cap-and-trade, there were other regulations in effect that impacted emissions. While the article began with a good observation here, it did not follow through. If the article had provided the exact regulations in effect, and the statistical impact on emissions, this would have been a more direct counter the statistics presented in the previous article, and the second article may well have won the argument.

Another topic mentioned by both arguments is the cost of cap-and-trade. The first article claims that cap-and-trade provides cost-effective, flexible compliance choices for regulated sources and minimized administrative costs. This argument is supported by the statistical citation that the overall costs of complying with the program will be only one fourth of what was originally predicted. Including the information about low costs for industry and government is very important evidence for the pro-cap-and-trade side. The low costs indicate that businesses will be more likely to want to comply with this program. The second argument brings up the same point about low cost, but claims that the low cost can actually result in perverse incentives, because a low price on emission permits reduces the incentive for companies to cut back their emissions. However, the critics’ argument fails for two reasons. First, the second article did not offer statistics to support this assertion. Second, the critic’s argument did not consider that the program’s low cost might have a positive incentive as well—encouraging companies to participate in the program at all. The point the second article was trying to make is that the negative incentives outweigh the positive ones, but the evidence is not strong enough to prove this point.

The final objection used by critics of cap-and-trade claims that expensive long-term changes will not be made if there is a cheaper source of carbon credits and that environmental protection needs better solutions. However, the statistics provided by the pro-cap-and-trade article indicate that these programs have been in place since 1976, so the changes reported by the statistics are definitely seen in the long term. As for “better solutions,” the critics’ article continues to make the same faults: it does not supply any data to support the objection. It does not even offer any specific programs that would be the kind of solutions that the critics advocate. The second article does raise an interesting point about how one solution can prevent a better solution. If the critics of cap-and-trade had managed to supply some facts to prove this point, the second article would have made the better argument.

Because of the lack of supportive facts or concrete details for the arguments in the second article, it is clear that the first article contains the better researched and supported argument. The first argument supplies specific, concrete data in support of the claim that cap-and-trade is effective. If the second article were to go beyond its vague warnings and use specific data in its objections, then the second article would have a more significant impact simply because the first argument does not seem to consider the possible drawbacks of the cap-and-trade structure. In that case, article two would win the argument.

Trait 1—Creation of Arguments and Use of Evidence

Score: 2

The introduction of this essay contains a position (“In this discussion of cap-and-trade, the proponents have made the better argument”) and then provides a developed explanation (“The pro cap-and-trade article then uses statistics to support its position. The second article uses mostly hypothetical assertions and warnings of what might happen, with no statistical data to support the position.”). The writer reinforces this claim with an analysis of the evidence for the arguments in the source text, first focusing on the strength of the proponents’ argument (“The first article then cites several statistics pointing to improvements in both SO2 and NOx. According to the article, improvements in emissions mean saved lives, and saving lives is very strong evidence for using cap-and-trade.”). The writer then moves on to the inadequacy of the opposing argument (“The article began with a good observation here, but it did not follow through.”). As further support, the writer introduces alternative interpretations of the evidence (“the second article did not consider that the program’s low cost might have a positive incentive as well”). Finally, the writer pinpoints the critics’ unsupported assertions about better solutions (“It does not even offer any specific programs that would be the kind of solutions that the critics advocate”). Overall, the response offers a well-developed, logical, and organized argument focused on the validity of the arguments in the source text. Therefore, Response B earns a score of 2 for Trait 1.

Trait 2—Development of Ideas and Organizational Structure

Score: 2

The response shows a clear structure, beginning with the importance of emissions control to frame the issue, then discussing points made by both sides of the argument, and finally concluding with an analysis. This structure permits a development of generally logical ideas that are sufficiently explored. The response establishes a clear connection between the main idea and supporting details within paragraph 2. The writer points to hypotheses about possible negative outcomes of cap-and-trade in paragraph 3 and then discusses why the claims are not supported. The writer goes on to explore the idea that the “concrete data in support of the claim that cap-and-trade is effective” outweighs the use of “vague warnings” by the opposition. The writer uses appropriate vocabulary and formal tone to express ideas, resulting in a response that is structured, focused, and developed. Therefore, Response B earns a score of 2 for Trait 2.

Trait 3—Clarity and Command of Standard English Conventions

Score: 2

This writer shows competency with several standard English rules, including subject-verb agreement, word usage, and the rules of capitalization and punctuation. The response offers a largely correct sentence structure, and the writer blends simple and complex sentences while maintaining clarity (“The second argument brings up the same point about low cost, but claims that the low cost can actually result in perverse incentives, because a low price on emission permits reduces the incentive for companies to cut back their emissions. However, this argument fails for two reasons. First, the second article did not offer statistics to support this assertion.”). Transitional words and phrases are used throughout (“in that case,” “however,” “while”). Overall, the response indicates a strong command of the English language and the level is appropriate for on-demand draft writing. Therefore, Response B earns a score of 2 for Trait 3.

Section 3

18. C

This language use question requires test takers to determine which definition of the word “unpretentious” matches its use in paragraph 1. Each of the answer options reflects an actual definition of the word “unpretentious,” so look closely at the way the word is used in context. The word is used as a part of the sentence “Plain, large and unpretentious, it described itself in an illuminated sign.” Here, the word “unpretentious” is part of a list of adjectives, beginning with the word “plain,” that describe the hotel mentioned in the previous sentence. So here the word “unpretentious” is used to mean “not fancy.” Choice (A) is incorrect, because the passage is not claiming that the hotel lacks ambition. Nor would a hotel be “easygoing,” so eliminate (B). Choice (C) is correct: “without excessive ornament” means “not fancy.” Finally, (D) is incorrect, because “straightforward, direct” is a characteristic that describes a person’s manner or an idea, not a building. The correct answer is (C).

19. C

This development question asks specifically how the nighttime view of the city (paragraph 2) affects the narrator. Consider all of the details in paragraph 2 to get a sense of the narrator’s attitude. The narrator uses phrases like “a city of beautiful streets and magnificent buildings,” “Every street or avenue glistens at night,” “another still more wonderful,” and “indescribable orgy of light.” Therefore, the author is impressed by the beauty of the city, especially the illumination. Choice (A) is incorrect. While the narrator does mention “a bewildering multitude of illuminated signs,” there is nothing to indicate that the narrator is overwhelmed. Eliminate (B) because, while the author does mention the number of lights in the phrase “tens of thousands of lights wonderfully displayed,” the phrase “wonderfully displayed” does not mean that the narrator is “wondering at” the number of lights. Rather, the narrator uses this phrase to express admiration for the signs. Choice (C) is correct: “rejoices” reflects all the positive language the narrator uses to describe the city at night. Choice (D) is incorrect, because the narrator is not “disillusioned” or disappointed by the city, but rather admires it. The correct answer is (C).

20. B

This structure question asks why the author chose to use a certain phrase, so locate the detail and consider the context of the surrounding text. The phrase occurs in paragraph 2: “The first impression of a stranger visiting a large American city at night is that he is in a children’s luminous palace.” The passage goes on to say “There are illuminations and decorations of every conceivable nature.” All of these descriptions refer to the “advertising” mentioned in the sentence, so the quotation in the question is referring to advertising signs, and “every conceivable nature” shows that the narrator is impressed by the number of signs. Choice (A) is incorrect: Although the use of the word “children” might seem to imply “innocence,” the author never observes “innocence” about the city. Choice (B) is correct, indicating that the author is impressed with how many advertisements there are in the city. Eliminate (C), because while the phrase “children’s luminous palace” is certainly a poetic representation of the narrator’s feelings, there is nothing about the “regal nature of America” anywhere in the excerpt. Choice (D) is incorrect because it is contrary to the author’s purpose: The author is admiring the nighttime city. The correct answer is (B).

21. A

This is actually a plot development question, because the phrase suggests something about the author’s broader experiences. Paragraph 3 states, “When I found myself finally back in my hotel I was to be the victim of still another disillusionment.” The passage continues “No country anywhere could rival America for hotels, I had thought.” So the “disillusionment” refers to the mistaken thought the author had about American hotels. “Still another” implies that the narrator has had mistaken thoughts about America before. Choice (A) is correct: This phrase indicates that the author has been mistaken before. Choice (B) contradicts the passage: All the details in paragraph 2 indicate that the narrator admires the city at night. Choice (C) is incorrect because the “National” is the hotel where the narrator is staying (paragraph 3), which reveals the mistaken assumption the author had about hotels. Choice (D) is incorrect because in paragraph 1 the author feels satisfied with the hotel (“I found a hotel that, from the outside, just suited my fancy,”) and so would not have been reluctant to return. The correct answer is (A).

22. D

This is a structure question about the use of “senile” in paragraph 3, so locate the detail and consider the context: “The furniture, consisting of one bed, one dressing-table, one wardrobe and one chair was obviously suffering from advanced senile decay.” This sentence occurs within a larger description of the shabby appearance of the room overall. Therefore, “senile” repeats the emphasis that all parts of the room are worn out, by emphasizing the age of the furniture. Choice (A) is incorrect: Although the use of the word is intended to mean “advanced age,” it applies to the furniture, not the hotel. Choice (B) is incorrect, because the author applies the word “senile” to all the furniture in the list—nothing in the room is new. Eliminate (C) because this paragraph discusses the room, not the manager. Choice (D) is correct: All the items of furniture are old, which builds on the author’s description of the room overall. The correct answer is (D).

23. D

This question asks about the character development of the narrator as shown through certain actions. Paragraph 3 directly states, “putting my revolver under the blanket near me in case of possible eventualities, I laid me down in peace to sleep.” The use of the phrase “possible eventualities” suggests that the narrator is preparing for anything that may happen, specifically because the door was “minus a lock for many a long day.” So, the narrator places the revolver in preparation for what may happen because of the unlocked door. Choice (A) is contradictory because “laid me down in peace” indicates that the author is not fearful, and the neighborhood is never described as rough. Choice (B) is incorrect: Even though the narrator states, “I was too tired, however, to bother,” the action of placing the revolver under the blanket is not due to tiredness, but a purposeful response to the unlocked door. Eliminate (C) because nothing in the excerpt indicates that the author has a naturally violent temper. Choice (D) correctly indicates that the narrator’s action is done in deliberate preparation for a possibly dangerous situation (the unlocked door). The correct answer is (D).

24. A

This question asks about the author’s purpose in choosing that sentence for the conclusion. So, consider the role the last sentence plays in the excerpt as a whole. Paragraph 5 relates the speech the narrator gives to the manager regarding the shabby hotel room: “never in ANY city at ANY time have I struck ANY hotel that for sheer rottenness compares with THIS one!” The manager would likely be outraged at hearing such a criticism, so for the excerpt to conclude, “I have an idea at the back of my mind that that manager-man doesn’t love Englishmen!” is an ironic understatement about the manager’s true feelings. Thus, (A) is correct. Choice (B) is incorrect because the purpose of the sentence is to close the scene with humor or irony, not guess about the manager’s preferences. Eliminate (C), because the narrator is angry about the condition of the room, not suspicious of the manager. Choice (D) is contradictory to the passage: The narrator does not show regret for complaining, but instead makes an ironic comment. The correct answer is (A).

25. B

This question asks about the narrator, so at first it may seem like a character development question. However, the entire passage is about the narrator, so use the main idea. A major theme is that the narrator is a foreigner visiting America, as is suggested by the title and the words “a stranger visiting a large American city” (paragraph 2). Choice (A) contradicts the passage: All the details in paragraph 2 indicate that the narrator admires the city. Choice (B) is correct since paragraph 6 indicates the narrator is from England. Eliminate (C) because the sentence “No country anywhere could rival America for hotels, I had thought” (paragraph 3) strongly indicates that the narrator has been to other places in America before. Even though the narrator shouts once in paragraph 5, there is no evidence to indicate that the narrator is loud-mouthed in general, so eliminate (D). The correct answer is (B).

26. This item asks the reader to identify statements that express the author’s purposes for writing this article. The two correct statements:

27. B

This structure question asks why the author uses the phrase “behind its hand” (paragraph 6). This phrase comes in the context of this sentence: “At a time when the one cry is ‘Production!’ the State adds (behind its hand), ‘Buy a Premium Bond, and let the other man produce for you.’” This sentence shows a contrast between the two actions of the state, almost a hypocrisy. Choice (A) is incorrect; this phrase doesn’t refer to the “no work at all” proposition in the previous sentence. Choice (B) is correct: This phrase is part of a contrast between the two actions of the state. Choice (C) is incorrect, because the analogy of the state to a human body is never made in this essay. Choice (D) is contradictory to the passage, because the author has a somewhat positive, rather than negative, view of gambling (paragraph 3). The correct answer is (B).

28. D

This structure question asks about the role that a detail—the image of the Monte Carlo gambler (paragraph 3)—plays in the passage as a whole. The author draws out this scenario to illustrate the aspects of “chance” and “game” that comprise “gambling” according to the dictionary definition (paragraph 2). Therefore, (A) is wrong; the author is not trying to make a statement about Monte Carlo as a place. Choice (B) is also incorrect; while “struggle” is mentioned, it is mentioned as a part of “game,” the aspect that the author is exploring. Choice (C) is contradictory to the author’s tone: The author has a generally neutral to positive view of gambling, so “reckless” is too strong. Choice (D) is correct: The image of the gambler is there to complete the picture begun with the definition of gambling. The correct answer is (D).

29. A

This structure question asks about what inference can be made from the sentence. The author asserts, “The case of the average man in favor of State lotteries is, quite simply, that he does not like Dr. Clifford.” The author implies that favoring lotteries is not based on logic but instead on personal dislike for Dr. Clifford. If the average man in favor of lotteries does not like the doctor (or his views), then it follows that the man is in favor of lotteries, simply because the doctor is not. Therefore, (A) is correct and (B) is wrong. Choice (C) is incorrect because intelligence is not addressed in the essay. Choice (D) is far too broad an assertion to make based on that one sentence—there’s no evidence that the average man would not favor anything represented by the doctor. The correct answer is (A).

30. D

This evaluation question asks about which detail supports the claim that people get pleasure out of games of skill and chance (paragraph 3). Immediately following this claim, the author draws out the scenario of the gambler in Monte Carlo. Choice (A) is incorrect because it is the bland description of the State Lottery. Choice (B) is incorrect; although it refers to making the country “contented” (paragraph 6), it doesn’t refer back to getting pleasure out of a game of chance. Choice (C) is incorrect; this quote (paragraph 2) just refers to the author’s consideration of the definition of “gambling.” Therefore, (D) is correct—the repeated use of “wonderful” indicates the gambler’s elation. The correct answer is (D).

31. B

This language use question asks the reader to examine the effect of the term “cold-blooded” in paragraph 5. The sentence reads, “and you simply take part in a cold-blooded attempt to acquire money without working for it.” Here the use of the term “cold-blooded” indicates a pragmatic pursuit of the lottery as opposed to the exciting language used to describe the Monte Carlo gambler’s experience. Therefore, (A) is incorrect—the author is depicting the players as passionless, not ruthless—and (B) is correct. Choice (C) contradicts the passage, since the author says gambling is not entirely evil (paragraph 4). Choice (D) is incorrect because this choice describes the consequences to society, rather than the experience of the individual in playing the lottery. The correct answer is (B).

32. D

This development question asks about how the author’s use of direct questions in paragraph 3 contributes to the author’s essay. Paragraph 3 states, “And is there any man who, having made a fortune at Monte Carlo, will admit that he owes it entirely to chance? Will he not rather attribute it to his wonderful system, or if not to that, at any rate to his wonderful nerve…?” These questions to the reader are rhetorical, designed to encourage thinking about the topic and convince the reader of the author’s perspective on gambling. Choice (A) is incorrect because there are no answers to these rhetorical questions. Choice (B) is contradictory to the passage, since the essay begins with a definition. Choice (C) is incorrect because the personality of the gambler is not under discussion. Choice (D) is correct—the questions cause the reader to consider the “game” or pleasure of gambling. The correct answer is (D).

33. A

This question asks about the author’s opinions regarding Premium Bonds. The author declares, “I am against Premium Bonds, but not for the popular reason. I am against them because there is so very little of the gamble about them” (paragraph 2). Thus, (A) is correct. Choice (B) is wrong because the author’s tone when discussing the redistribution of funds (paragraph 6) is definitely satirical: “Instead of a million ten-pound citizens, we should have a thousand ten-thousand-pounders and 999,000 with nothing. That would be the official way of making the country happy.” Choice (C) is contradictory because the author feels that Premium Bonds “have so very little of the gamble about them.” Choice (D) is incorrect because the author mentions “hypocritical” in paragraph 1 as the views of others, not his own view. The correct answer is (A).

34. B

This point of view question asks about the author’s tone toward the reader, so examine the passage for tone indicators. In several places the author gives opinions and addresses the reader directly: “There are many occupations—and many jobs— that you would enjoy” (paragraph 3), “You need not restrict your job search to careers related to your degree” paragraph (7), and “It’s never too late to change careers” (paragraph 1). Overall, the author offers encouragement to the reader or job seeker. Choice (A) is incorrect, because “condescension” means talking down to someone, which is not what the author is doing. Choice (B) is correct: The author is offering information to reassure and help job seekers. Choice (C) is incorrect, because the author is not trying to intimidate the reader. Choice (D) is incorrect because “satisfaction” expresses a concern with the self. However, this author is concerned primarily with the reader. The correct answer is (B).

35. A

This question asks about the author’s primary purpose in writing the article. The author’s identification as part of the Department of Labor suggests that the author is familiar with the realities of the working world, and paragraph 3 asserts “There are many occupations—and many jobs—that you would enjoy.” The final paragraph, paragraph 11, states, “It’s never too late to change careers.” Therefore, the author is trying to help people persist in a job search despite temporary setbacks. Choice (A) is correct, because the author emphasizes “keeping your options open” (paragraph 3) and “learning a variety of tasks helps you to sharpen abilities” (paragraph 5). Eliminate (B) because the author’s tone is encouraging, not cautionary. Choice (C) is incorrect, because while the article mentions low grades and work history gaps, it does not mention the specific types of jobs that accept people with these limitations. Choice (D) contradicts the passage: Paragraph 5 states, “no one job uses all your talents.”

36. D

This structure question asks how this sentence relates to the article as a whole. The sentence “Consider that electrical technicians are now repairing fuel cells, for example, or that veterinary technicians become pharmaceutical sales workers.” This sentence appears in the context of a paragraph that begins with “You need not restrict your job search to careers related to your degree,” (paragraph 7). Therefore, the sentence in question offers support for that topic sentence. Choice (A) is incorrect: While the sentence does present examples of specific careers, the author is not advocating that readers pursue these careers. Eliminate (B) because it contradicts the topic sentence of the paragraph. Choice (C) may seem attractive, because the last sentence of paragraph 7 reads “Often, technical skills are applicable to many settings.” However, the purpose of paragraph 7 is to address the flexibility of skills in general, not just technical skills, so (D) is correct.

37. C

This purpose question asks you to analyze the author’s rhetorical technique of question-and-answer in paragraph 9. The topic sentence of this paragraph states, “People overcome all kinds of challenges to find satisfying work.” The paragraph then goes on to raise the same challenges that were brought up in paragraph 8 and offer solutions to those challenges one by one. Choice (A) is incorrect because the paragraph does not address which challenges are “worse” or not. Choice (B) is incorrect because paragraph 9 is a response to paragraph 8 (the previous paragraph), not support for the next paragraph. Choice (C) is correct: Paragraph 9 uses the question-and-answer format to address the concerns raised in paragraph 8. Choice (D) is incorrect because the question-and-answer format does not help the author’s point by raising the concerns; it helps the author’s point by offering solutions for those concerns. The correct answer is (C).

38. D

This language use item requires test takers to determine which definition of the word “overcome” matches its use in paragraph 9. Each of the answer options reflects an actual definition of the word “overcome,” so look closely at the way the word is used in context. The word is used as a part of this sentence: “People overcome all kinds of challenges to find satisfying work,” which is connected to the later phrase “how you handle adversity.” So here, the word “overcome” is used to mean “rise above” the challenges or the adversity. Choice (A) is incorrect, because “overpower” means to “defeat in physical contest” or “overwhelm the senses”; neither makes sense with “challenges.” Choice (B) is also incorrect, because “shock the challenges” would not make sense either. Choice (C) might seem attractive because of the phrase “a good indicator of your ability to persevere”; however, “People outlive all kinds of challenges” would not make sense in a paragraph advising job seekers to assertively handle adversity. Choice (D) is correct: “surmount” means “rise above.” The correct answer is (D).

39. A

This question asks you to make an inference about the “workers” mentioned in the quotation. To answer this question, refer to the paragraph mentioned and read beyond the sentence to find out more about the “workers.” The topic sentence of paragraph 11 is “It’s never too late to change careers” followed by examples of workers who move into very different fields. Therefore, (A) is correct. Choice (B) contradicts the passage, which states, “For each of these workers, the desire for job satisfaction outweighed the desire for status quo.” Eliminate (C) because “status quo” means “current state” not “prestige.” Choice (D) is incorrect because the workers in the example did not all return to school—only one did. The correct answer is (A).

40. This item asks the reader to choose two examples the author uses to support the claim that prospective employees can take steps to better their odds of employment. Answer (a) is wrong because the author advocates working to gain skills as a remedy for lack of experience. Answer (c) is wrong because it contradicts paragraph 6. Answer (d) is wrong because author advocates a focus on skills rather than chronology to address gaps in work history (paragraph 9). The two correct answers are as follows:

41. C

This is a purpose question about the author’s intended audience, so use the main idea from the passage as a starting point. This author is concerned with correcting “career myths” so that these myths won’t “derail a career search and sap motivation” (paragraph 1). Therefore, the author is addressing job seekers who might be affected or “derailed” by these myths. Choices (A) and (B) are incorrect: There is nothing in the article to indicate that the information is targeted specifically at first-time job seekers or college students. Choice (C) is correct: The article addresses myths that cause people to doubt their ability to get a job (paragraphs 8 and 10). Choice (D) is incorrect because addressing work history gaps is just one detail in the passage, not the purpose of the whole article. The correct answer is (C).

42. C

This character development question asks specifically how her father’s words affect young Susan. After the sentence “It would never do to have a woman overseer in the mill,” (paragraph 2), the very next sentence conveys Susan’s reaction: “This answer did not satisfy Susan and she often thought about it” (paragraph 3). Choice (A) is incorrect: As she watched the mill workers, the fact that women weren’t overseers “continued to puzzle Susan” (paragraph 3). Eliminate (B) because, while Susan questions her father before being told “it would never do to have a woman overseer,” there is no indication that Susan questions her father’s judgment after being told this. Choice (C) is correct: Susan sees that her father’s words do not match what she observes happening in the mill. Choice (D) is incorrect because nothing in this excerpt indicates that Susan goes to work in the mill or rebels against her father. The correct answer is (C).

43. A

This structure question asks about how the details about Susan’s home life enhance the narrative, so connect these details to the main idea. The main idea of the passage is about the kinds of work considered appropriate for women. The description starts with the second sentence of paragraph 5, indicated by “in the home of the millowner.” At home, Susan’s mother cared for three daughters and eleven boarders, and that her days were filled with cooking, washing, ironing, mending, and spinning. Then the passage states, “But she was capable and strong and was doing only what all women in this new country were expected to do.” So, the description of Susan’s home life shows both her mother’s capabilities and the enormous workload women were expected to handle. Therefore, Choice (A) is correct. Choice (B) is incorrect because the passage does not indicate that Susan’s father is absent; rather, her mother’s workload is depicted as a typical amount of daily duties. Choice (C) is incorrect because the housework and the millwork are not compared in the passage. Even though boarding the millworkers is part of Susan’s mother’s work, it is only one part of it, not the primary reason to give all the details of home life. Therefore, eliminate (D). The correct answer is (A).

44. D

This evaluation question asks which sentence from the passage supports the idea that Susan is learning skills that enhance independence. Since several answers contain quotations related to the major themes of women’s work or independence, consider what is implied as well as what is directly stated. Choices (A) and (B) are incorrect, since these sentences refer to the women who work in the mills, not Susan. Choice (C) is incorrect; even though this quotation describes Susan, she is only observing other workers, not learning skills. Choice (D) is correct: It is part of the sentence that states, “Susan, even before she was six, was very useful” (paragraph 5), so this quotation highlights Susan’s ability to care for herself and do grown-up work. The correct answer is (D).

45. B

This development question asks about the character of Sally Ann as shown through the overseer’s actions. The reference is located in paragraph 3: “When the yarn on the beam was tangled or there was something wrong with the machinery, Elijah, the overseer, always called out to Sally Ann.” The passage goes on to say “Sally Ann never failed to locate the trouble or to untangle the yarn.” This shows that Sally Ann’s mechanical ability was known and relied upon. Choice (A) is incorrect, because the overseer is not relying upon Sally Ann’s weaving abilities in these sentences, nor does the passage indicate that she is a more efficient weaver than the other women. Choice (B) is correct: The overseer (another worker) recognizes her mechanical ability. Choice (C) is somewhat contradictory since Sally Ann is the person who untangles the yarn. Choice (D) can be eliminated because there is no information about Sally Ann’s hopes or whether she enjoys being asked for help. The correct answer is (B).

46. A

This question asks about Susan’s father’s attitude about the cotton mills. Susan’s father’s point of view is a major theme of the passage: He has definite ideas about the women who work in his mills. Since the cotton mills are discussed throughout most of the excerpt, use the passage to either confirm or eliminate answers based on details in the choices. That the mills were an acceptable place for women to work is not directly stated in the passage but is strongly implied by the fact that Susan’s father employs women weavers, so keep (A) and eliminate (B), since that answer directly contradicts the fact that Susan’s father employed women. Choice (C) can be eliminated because even though the passage states, “Brides were proud to display a few cotton sheets instead of commonplace homespun linen” (paragraph 4), there is no indication in the passage that Susan’s father felt cotton would replace homespun entirely. Eliminate (D), because even though paragraph 3 states the mill had a “general air of efficiency,” there is no indication that Susan’s father considered men generally more efficient than women. The correct answer is (A).

47. This character development item asks the reader to identify three adjectives that accurately describe Susan as she watches the mill workers. This scene is found in paragraph 3. Here are the three correct adjectives:

48. D

This question asks about the mill that Susan’s father built. Since different details about cotton mills are discussed in paragraphs 3, 4, and 5, use the passage to either confirm or eliminate answers based on details in the choices. Choice (A) is incorrect: The first sentence of paragraph 5 directly states that cotton had to be brought from Troy to the mill, a distance of 40 miles. Choice (B) is incorrect, because paragraph 5 states that the workers boarded at the home of the millowner, not the mill. Eliminate (C) because the passage states that Susan was born in 1820 (paragraph 4) and Susan’s father built the mill when she was two years old (paragraph 5). Choice (D) is correct: Paragraph 5 states that Susan’s father’s mill was built beside a brook, and paragraph 4 states that “men like her father, Daniel Anthony, saw a potential cotton mill by the side of every rushing brook,” implying that a source of running water is necessary for a mill. The correct answer is (D).

49. B

This development question asks about the women workers in Susan’s father’s mill. The women mill workers are shown at work in paragraph 3, their backgrounds are discussed in paragraph 4, and their living conditions are mentioned in paragraph 5. Choice (A) is incorrect, because only Sally Ann is presented as a worker possibly qualified to be overseer—there is no information about the others. Choice (B) is correct: It accurately summarizes the sentence in paragraph 4: “young women, eager to earn the first money they could call their own, were leaving the farms, for a few months at least, to work in the mills.” Choice (C) directly contradicts the passage, which states in paragraph 5 that the workers lived at the home of the millowner. Eliminate (D) because Susan’s mother wasn’t helped by the mill workers but by “a thirteen-year-old girl who worked for her after school hours,” (paragraph 5). The correct answer is (B).

50. The option that correctly completes the sentence for each “Select” option is as follows:

Drop-Down Item 1: Verb tense and Misplaced modifier/illogical word order

Option 2 is correct: In this sentence, the verb “had” (the simple past tense of “have”) is used appropriately in a sentence that refers to a period in the past.

Option 1 is incorrect, because the present tense “I see him developing” does not fit in this sentence about a “a period of six years ending in June 2009.” Past tense is needed here. Option 3 is incorrect: This option results in illogical construction: “I had great pleasure from a paste-up assistant at the beginning, into a fully functioning Graphics Design Project Coordinator.” Option 4 is incorrect, because this option results in ambiguous construction that could be read “with great pleasure, he was developing” instead of “As I saw him, with great pleasure.”

Drop-Down Item 2: Parallelism and run-ons

Option 3 is correct: This option correctly divides two independent clauses with a period and keeps the items in the list in parallel adverbial form: “quickly and effectively.”

Option 1 is incorrect: A comma is too weak to divide the two independent clauses and so this option results in a comma splice. Option 2 incorrectly merges “quickly, effectively, and during” into a list form, resulting in a comma splice as the two independent clauses are connected with a comma. Option 4 is incorrect because the items in the list (connected by “and”) are not parallel: “quick” is an adjective and doesn’t match the adverb “effectively.”

Drop-Down Item 3: Parallelism

Option 4 is correct: This option keeps the structure parallel by pairing an infinitive with an infinitive in a comparison structure: “just as able to give guidance as to step.”

Option 1 is incorrect because it results in the construction “able to give guidance and step back and take direction” which is a run-on structure. A list of three items should be written “give guidance, step back, and take direction.” Option 2 is incorrect, because the dependent clause is separated from the rest of the sentence with a period, forming the sentence fragment “Just as able to give guidance and step back and take direction when another is leading the group.” Option 3 is incorrect because the construction “to give guidance and stepping” is not parallel, as it pairs an infinitive with a gerund.

Drop-Down Item 4: Misplaced modifiers or illogical word order

Option 4 is correct: In this sentence, the words “still” and “fondly” are correctly placed next to “remembered,” which they both modify. The word “even” is placed next to “a year” to emphasize the time period, so that the sense of “still remembered” matches “even a year.”

Option 1 is incorrect: In this order, the placement of the word “fondly” makes it appear to modify the word “here” when it should modify “remembered.” Option 2 is incorrect: In this option, the placement of the word “even” makes it appear to modify “he,” resulting in the phrase “even he” which distinguishes a person from other people. However, since no other people are the subject of discussion, “even he” is unnecessary and confusing. The word “even” should modify “year,” to distinguish a time period. Option 3 is incorrect: The placement of the word “still” seems to modify “fondly” when it should modify “remembered” so that the emphasis on time in “still remembered” would match “even a year.”

MATHEMATICAL REASONING

Part 1

1. D

Replace every z with –3 and solve the equation. Remember that when you square a negative number, it becomes positive, but when you cube a negative number, it stays negative. So,

image = image = image = image= –3

2. C

To calculate the minimum amount of music (7.2 hours), divide 3.6 by 2 and then multiply the result by 4. To calculate the maximum amount (14.4 hours), multiply 3.6 by 4. The total must be between these numbers. If you chose (D), you gave the range for a single CD instead of for all 4.

3. D

The total time must include the trip there, the 3 hours in between, and the trip back. To calculate the time for the trip there (2.5 hours), divide 145 by 58. The trip back is also 2.5 hours, so for total time add 2.5 + 2.5 + 3.

4. B

The July temperature should equal four times the February temperature, so 4t (remember that t represents the February temperature) is equal to 82. If you chose (C) or (D), you solved for the February temperature and got 21 and incorrectly made that a part of the equation. Note that the question asks for the formula not the solution.

5. 38,000

First calculate the number of tomatoes that were crushed (20% of 50,000 = 10,000 tomatoes) and subtract this number from the original 50,000. Now calculate 5% of the remaining tomatoes, keeping in mind that there are 40,000 tomatoes now not 50,000. So 5% of 40,000 = 2,000 tomatoes. Subtracting from 40,000 leaves 38,000 tomatoes.

Part 2

6. A

The total number of workers employed in neither clerical nor professional jobs is equal to the total number of workers (2,300,000) minus the number of clerical workers (19% of 2,300,000 = 437,000) minus the number of professional workers (24% of 2,300,000 = 552,000). If you answered (C), you found the total workers in those two professions but forgot to subtract it from the total workforce

7. B

This question can be solved algebraically or by plugging in answer choices for Miriam and seeing which number works. To solve algebraically, set Betty’s stamps equal to m – 6. Now, m + m – 6 = 2m – 6 = 42. Solving, m = 24.

8.

image

Since point P is at –5 and point S is at 3, P + S = –2.

9.

image

Since point Q is at –3, point R is at 1, and point T is at 4, |Q – R| – T = |(–3) – 1| – 4 = |–4| – 4 = 4 – 4 = 0. Don’t forget to do the operation inside the absolute value sign before applying the absolute value.

10. A

To solve this equation, set up the rate of erosion as a ratio of distance divided by time. Since the rate of erosion remains constant, this ratio will remain constant. Therefore the original distance divided by the original time image is equal to the new distance divided by the new time image.

11. D

The question asks how many times stronger lens X is than lens Y, so divide the strength of lens X by the strength of lens Y. 3 × 105 ÷ 6 × 102 = 300,000 ÷ 600 = 500. If you chose (C), remember that 500% of a number is equivalent to multiplying by 5 not by 500. If you chose (A) or (B), you probably divided 6 by 3 instead of 3 by 6.

12. C

One good way to solve this equation is with ratios. Mark’s rate of mowing, which is equal to the amount mowed divided by time, remains constant. So image = image. Solve this equation using cross multiplication to determine that x = image.

13. B

First you must recognize that the front and the back of the box are the smallest two sides, since they have the smallest dimensions (2.5 × 7). So now, using the surface area formula, add up the other 4 sides and leave off the front and the back. The perimeter of the base is 2.5 + 2.5 + 7 + 7 = 19, so surface area 19 × h = 19 × 10 = 190.

14. −54

First substitute the values of x and a into the equation and then solve, being careful about order of operations. 3x(x – 2a)x = 3(–2)(–2 – 2 × (.5))2 = (–6)(–2 – 1)2 = (–6)(–3)2 = (–6) × –54, if x = –2 and a = 0.5.

15. 9, 34

The task here is to find a value in the first pull-down menu that will give a perimeter found in the second pull-down menu. Just start plugging in values from the first menu until you get a perimeter that is in the second menu. If x = 9, then perimeter = 9 + 9 + 16 = 34.

16. 2,550

The laser printer and hard drive do not change price so they will still cost $650 and $250 respectively. The price of the computer, however, goes up 10%, so calculate 10% of $1,500 and add that to the previous price. So now the computer costs $1,500 + $150 = $1,650. The total bill therefore would be $1,650 + 650 + 250 = $2,550.

17. A

This is a hard algebra problem but an easy Backsolving problem. Always start with a middle answer choice, such as (C). Say there are 26 women at Universal. If there were twice as many women as men, that means there would be 13 men. Altogether, there are supposed to be 78 employees, but your numbers add up to only 39. Could (C) be the right answer? Nope, too small. You need a bigger number. Move up to (B) and say there are 42 women, which means there would be 21 men. You’re supposed to get 78 employees, but 42 + 21 equals only 53 employees. Still too small. That must mean the correct answer is (A).

18. image or image

Probability questions always ask for a part divided by a whole. To calculate the probability that a random bird is not a duck, you need to divide the total number of non-ducks by the total number of birds. The total number of non-ducks is equal to the number of geese plus the number of robins.

19. D

This question tests distribution. Be sure to distribute the term outside the parentheses to both of the terms inside the parentheses, and then combine like terms and simplify. 2a(a – 3b2) + a2 = 2a2 – 6ab2 + a2 = 3a2 – 6ab2. But this is not an answer choice. If you chose (A), you forgot to add the a2. If you chose (B), you did not distribute the 2a. Notice that both terms now are divisible by 3a, so you can get (D) by pulling 3a out front. You may also try plugging in for values of a and b in the original equation and checking the answer choices to see which is equivalent.

20. D

Use the slope equation to find the missing point. image = m (slope). Since the slope of the line is given as 3, therefore, you find that 3 = image = image = image. Multiply both sides by the denominator, so 3(x + 2) = 21, and x = 5.

21. B

To write the equation of a line in the form y = mx + b, you need to know the slope m (here given as 3) and the y-intercept b, which is not given. To calculate b, plug in the point on the line that you know (–2, –10) and solve. So –10 = 3(–2) + b. Solving, b = –4.

22. B

The answer choices should clue you in that this is a setup problem: There is no need to complete the algebra. To find out how much the typist is earning per hour, divide $65.40 (his daily pay) by 4 (the number of hours he is currently working). This is the typist’s hourly rate. Now, multiply that by 7 hours to find out what he would be paid if he worked 7 hours a day: 7 × $65.04 ÷ 4. The correct answer is (B).

23. Action-Adventure 18, Drama 17. The total of the values on the chart must add up to 100%, since the chart represents all the people surveyed. The current values add up to 65. Subtract this from 100 to get 35, which should be the sum of the missing numbers. Looking at the length of the bars on the chart, you can see that 10 is much too small and that 25 is too large. Try numbers until you find two that add up to 35. The only options are 17 and 18. Since Action-Adventure is a longer bar than Drama, place the 18 by Action-Adventure and the 17 by Drama.

24. A

The ratio of Comedy to Other Types is 25:10, but this is not one of the answer choices. You must reduce the ratio just like you reduce a fraction. Divide both 25 and 10 by 5 and you get 5: 2.

25. D

The question asks for an average price, so you need to divide the total price of the tickets by the total number of tickets. The total price of the first-class tickets is 110(6.00), the total price of the second-class tickets is 172(3.50), and the total number of tickets is 110 + 172.

26. image or 2.25

Be careful about order of operations. First multiply both sides by 2 to eliminate the fraction. So (4x + 3)2 = 144. Take the square root of both sides. 4x + 3 = 12 and 4x = 9.

27. C

The post office is located halfway between points A and E. Using the diagram, we can calculate that the distance between A and E is 23 miles. Therefore, the post office is located 11.5 miles from A and 11.5 miles from E, right in the middle. Start at point A and go 11.5 miles to the right. You will be between B and C.

28. C

First add the tip to the given bill, so multiply $36 by 20% and then add $36. So the total bill is $43.20. Now divide this by 4.

29.

image

Remember that when a point lies on the y-axis, that means the x-coordinate is 0, and when a point lies on the x-axis, the y-coordinate is 0. Given the equation, y = imagex + 3, when x is 0 then y is 3. So mark the point (0, 3) on the y-axis. Then set y = 0 in the equation and solve for x. imagex + 3 = 0, so imagex = − 3. amd x = –6. So mark point (–6, 0) on the x-axis.

30. 200

The classic mistake here is to find 60 percent of $120, or $72. However, the $120 is 60 percent of Karen’s entire paycheck. In other words, you need to find 60 percent of a number we don’t know yet, and set that equal to $120. Since $120 is 60 percent of the paycheck, this means that 120 = imagex. Solving for x, the answer is 200.

31. C

In this problem, there was more information given than you really needed. After one day, the amount of water in the reservoir had doubled from its original amount. After five days, the amount of water had quadrupled from its original amount. If all we need to know is how much water there was after five days, the fact that it doubled after the first day is irrelevant. If x represents the original amount, the correct answer is 4x.

32. D

There are 4 chefs, and each one averages c per month. There are 18 servers, and each one averages $840 less than the chefs’ wages (c). Subtract $840 from c and multiply that by 18 servers. Add the totals of the two groups to get the monthly payroll.

33. B

First calculate the area of the new sign using a ratio of area to price. image = image. Cross multiply and solve for x = 96. So the area of the large sign is 96. Now divide area by height to get the length = 12.

34. B

Since the total number of games team A will play is 44, take 75% of 44 to calculate the total number of games that team A must win. 75% of 44 is 33 games, so that is their target. So far team A has won 28 games, so they need to win an additional 5 games.

35. D

image

In order to find the area of a more complicated figure, divide the figure into more manageable shapes. Here if you draw in the dotted line shown, you will have two rectangles. The area of the larger rectangle is 15 × 11 = 165. The area of the smaller rectangle is 3 × 6 = 18. So the area of the whole figure is 165 + 18 = 183.

36. 4.5

To find the empty space, find the volume of the entire chest by multiplying the three dimensions (length, width, depth): 2.5 × 1.5 × 2 = 7.5 cubic feet. Then subtract the size of the blankets (3 total cubic feet, since 3 blankets) to get 4.5 cubic feet of empty space.

37. A

After Michelle paid the deductible of $500, there was $350 remaining. The insurance company paid 80% of that, so $350 × 0.8 = $280. Michelle had to pay the remaining $70 ($350 – $280 = $70).

38. The volume of cylinder L is [less than half] the volume of cylinder R.

Use the formula for the volume of a cylinder to calculate the volume for each cylinder. Although the numbers in both cylinders (8 and 20) are the same, the volumes are not equal. The reason is that the formula for volume requires that the radius of a cylinder be squared. Therefore, the radius has a greater effect on volume than height does. As a result, the volume of the cylinder with the larger radius (20) has a greater volume. Volume of cylinder L = πr2h = π42 × 20 = 320π. Volume of cylinder R = π102 × 8 = 800π. So the second cylinder is much larger in volume.

39. A

First calculate the volume for each cylinder L = πr2h = π42 × 20 = 320π. So this is the volume of the water poured into cylinder R. The water in cylinder R now forms a cylinder that comes part of the way up the side of cylinder R. To determine the height, use the volume equation again: 320π = πr2h = π102h. So 320 = 102 × h and h (the height of the water) = 3.2.

40. 92,650

To calculate the minimum revenue, consider which empty seats will cost the airline the most money. 3 empty first-class seats will reduce the total revenue much more than 3 empty coach seats. So calculate the total income from (30 − 3) = 27 first-class passengers and (325 − 30) = 295 coach passengers. So Total minimum revenue = 27 × $700 + 295 × $250 = $18,900 + $73,750 = $92,650.

41.

image

The distance between two points on a number line is found simply by subtracting (and distance is always positive). So 5 – (–7) = 12. × 12 = 4 so you should mark 4 on the number line.

42.

image

Insert the values from the number line into the equation and solve. ((–5) – (–2)) – 2 = (–3) – 2 = –5. Remember that subtracting a negative number will move to the right on a number line and subtracting a positive number will move to the left.

43.

image

When two triangles are similar, respective sides must have the same proportion. So in this case, the ratio of the height to the length must be 8:10. Look for two numbers among those available that give the same ratio. 4 and 5 are the only possibilities. Note that image reduces to image.

44. 10

Count ’em up! On Monday, he takes 2 tablets: one at 2 p.m. and then one 8 hours later at 10 p.m. On Tuesday and Wednesday, he takes 3 tablets a day, at 6 a.m., 2 p.m., and 10 p.m. On Thursday, he takes 2 tablets: one at 6 a.m. and the last one at 2 p.m. 2 + 3 + 3 + 2 = 10 tablets.

45. C

Notice that the question says her investment increased by 180%. That means that the difference between the new value and the old is equal to 180% of $14,000. 180% of $14,000 is $25,200. Add this to the original ($14,000) to get $39,200—the current value of her investment.

46. A

Diameter AB has length 8, so the radius of the circle is 4. Since AB is a semicircle, the center of the circle must be directly between points A and B at point (2,7). Point C, then, must be directly to the right of the center, since it is the midpoint of the arc. Therefore, its x-coordinate will also be 6 (since the radius is 4) and its y-coordinate will be 7.

SOCIAL STUDIES

1. B

This EXCEPT question is asking you to decide which answer choice CANNOT be used to deny a child employment. Each of the choices is mentioned in the passage as a possible reason to keep children from working—except for one: (B), gender. Age is mentioned in the second half of the passage as a restrictor to employment, so eliminate (A). As for (C), the actual phrase “hazard levels” is not in the passage, but the third sentence from the end says, “as long as the job is not determined to be dangerous or ruinous to their health or well-being,” which basically means the same thing. This sentence also rules out (D). Only gender is NOT discussed as a reason to deny a child employment.

2. B

Choice (A) says that it should be up to kids to decide when and whether they want to take a job—but the passage says this is decided by the Department of Labor, so you can eliminate this one. Choice (C) says kids can’t work under any circumstances, which directly contradicts the entire passage, so you can eliminate this one, too. Choice (D) is saying the same thing as (C) using different words. The correct answer, which sums up the reason for the need for regulations, is (B).

3. B

Who would be likely to support regulations to protect children? Factory owners may be kindhearted, but then again, there have been a lot of factory owners who have exploited child labor—so eliminate (A). Child protection agencies would seem very likely to protect children; let’s hold onto (B) while we look at the other choices. Worker’s unions might well also support regulations to protect children, so let’s hold onto (C) as well. Small companies and agricultural workers might be kindhearted too, but we don’t know that they will be, so eliminate (D). We have two choices left: (B) and (C). Which is better? The correct answer is (B).

4. D

The passage states that the Berlin Airlift was “in response to Soviet Premier Joseph Stalin’s attempt to block supplies to Berliners.” This is closest to (D). Choice (C) is tempting, but the creation of the Warsaw Pact happened after the Berlin Airlift. Choice (A) is unsupported by the timeline and (B) does not work, since if the Marshall Plan had been completely successful, Berliners would not have needed assistance.

5. A

According to the timeline, the Warsaw Pact was created after the formation of NATO. All remaining answers include events that happened before NATO.

6. B

The American flag is located next to the lower-left, southwestern region of the map. Choice (A) is not true, since this region is smaller than the eastern region. Great Britain controlled the west, not the east, so (C) is wrong. Choice (D) is incorrect, since Stalin is Soviet and thus controlled much of eastern Germany.

7. D

In the passage, Grant says that the Civil War would change the nation as a whole. Which historical development best supports that? The triumph by one side in a civil war isn’t an example of how the country was changed by the war. Eliminate (A). The entry of the United States into World War I does not seem to be directly related to the Civil War either. Eliminate (B). Women’s winning the right to vote also doesn’t seem like a direct result of the Civil War. Eliminate (C). The correct answer is (D). The Civil War was waged to free the African American slaves—which profoundly changed the entire nation.

8. D

You may have been tempted to pick (A) because it concerned taxation, but the principle of “taxation without representation” means that a government doesn’t have the right to tax its citizens unless those citizens have the right to elect the government and thus have a say in how much they will be taxed. This was one of the principles that drove the 13 colonies to declare independence from England in 1776. Choices (B) and (C) have nothing to do with the Twenty-fourth Amendment as outlined in the question. Only (D) does: Just because a person doesn’t pay his taxes does not mean his right to vote can be taken away.

9. C

The passage says that the young men went west in search of opportunities such as finding gold and striking it rich. Which of the answer choices says that? The correct answer is (C). Choices (A), (B), and (D) didn’t appear in the passage itself—and while these all might have been reasons for someone to head west, the correct answer to a GED® Reading passage is almost always going to come from within the passage.

10. A

The only reference to towns in the passage comes in the second sentence, and it refers to San Francisco and Monterey. The next sentence reads, “There they found jobs….” So why did people settle in those towns? The correct answer is (A). You might have been tempted by (C) because it refers to Monterey and the gold to be found there—but by the time they got to these towns, the passage says that the settlers had given up on being prospectors and just needed jobs. Choices (B) and (D) contain information not found in the passage itself.

11. C

The First Amendment protects five essential freedoms, three of which are listed in (C), the correct answer. The Second Amendment protects the right to bear arms (A). (D) is obviously wrong: The primary role of Congress is to make laws.

12. A

The First Amendment does not guarantee the right to vote. Rather it makes five guarantees: those listed in (B) through (D) and also the freedom of assembly.

13. D

Before you turn to the question, make sure that you understand the pie chart. The title says that it is about how children are being educated. It says that 39.2 percent are being educated in public school, 31.5 percent are being educated in private school, and the rest of the children are being educated in other ways. Which statement is supported by the information in the pie chart? Choices (A) and (B) imply value judgments that are not supported by the chart. Who knows which is better, or which ones the children prefer? Choice (C) contradicts what we can see from the chart. If tutoring at home were the most popular method, it would have the largest percentage of kids doing it. The correct answer is (D).

14. D

This was a tough question, because it asked you to do some analysis. A pure market economy, according to the passage, is one in which the forces of the market control what happens; in other words, there is no government control whatsoever. So which of the answer choices would NOT be found in a pure market economy? Monopolies could occur in a pure market economy because there would be no government rules to prevent them. Small businesses would probably flourish—at least until monopolies put them out of business. There would certainly be unemployment because the markets would hire people only when they needed them. And presumably, stocks and bonds (which provide capital for new businesses) would be found in large numbers. The only things you would NOT find in a pure market economy, as unlikely as it seems, would be taxes and welfare (both of which are controlled by the government). The answer to this difficult question is (D).

15. D

The passage says that in a socialist economy, all the money is distributed equally among the members of a society. So which of the groups in the answer choices would benefit the least from the switch to a socialist economy? Choices (B) and (C) would benefit quite a bit, because these are relatively poor people who would get a bigger slice of the pie than they have now. Eliminate (B) and (C). Between the two choices we have left, who benefits the least? The correct answer is (D), highly skilled labor, because presumably they were being well paid for their unique skills, and would lose money in the redistribution of wealth. Government employees in socialist systems usually tend to gain power because they get to decide who receives the wealth.

16. D

As you study the photograph, you will probably notice two things: (1) The car in the photo is quite old, and (2) it isn’t working. The people in this old photograph are pushing the car instead of riding in it. The question itself says that while today we sometimes have difficulties with cars, the people back then had to deal with another kind of problem. What was it? Does it seem as if the people in that photo are concerned about pollution? We didn’t think so. Are they concerned about seatbelts? Not when most of them are pushing the car. Are they concerned about heavy cars? Now, we’re getting warmer. If you were pushing a car, you would probably want it to be as light as possible. But let’s read the last answer choice before we pick: Could the people in the photo be concerned with a shortage of gas stations? Aha! This answer gets to the heart of why they are pushing the car in the first place: It is out of gas. The correct answer to this question is (D).

17. C

Religion, factories, and lynchings are simply not mentioned anywhere in the passage, so eliminate all but (C).

18. D

Read the graph carefully before looking at the questions. We see bars representing men and women who take the SAT, broken down by ethnic group. From this graph, we can see that in all but one group, more women than men take the SAT. The one group where this does not appear to be true is that of Asian Americans. Looking at the answers, (A) makes a claim not supported by the graph. We are not given any information about how men and women perform on the test—just the fact that they took it. Choices (B) and (C) make statements of fact not supported by the graph. We have no idea whether numbers are declining or increasing from year to year. This graph is of a single snapshot in time. The correct answer is (D): According to the chart, clearly more women than men take the SAT.

19. C

The question is basically asking, “Which of the following is NOT an example of a plutocratic government?” The best answer is (C), in which the leaders are elected by popular vote. In a plutocracy, the leaders are chosen because they are the wealthiest citizens. This means that there is no vote. All the other choices reflect situations that could occur in a plutocracy.

20. D

You don’t have to know anything about Andrew Mellon to realize that, in this statement at least, he is on the side of the people. He writes that “unwise” public officials “fall to these interests of monopoly as against the welfare of the people of whom they are supposed to serve.” So we are looking for an answer choice that is sympathetic to the people and critical of officials who lose sight of the people’s welfare. Choice (A) seems possible at first, but goes too far: Mellon doesn’t suggest that public officials who profit from a monopoly shouldn’t run for office. Choice (B) is wrong because it sides with the public officials over the interests of the people. Choice (C) goes too far, because Mellon never says the profits of monopolies should be shared by all. This leaves us with (D), which is a simple restatement of what Mellon says in the passage.

21. B

The passage compares the relationship between the federal and state governments to, respectively, the hub and spokes of a wagon wheel. This is called an analogy (which means a comparison between two things to help you understand one of them better). You have to find an answer choice that illustrates the analogy. Choice (A) is no good because, in the analogy, the entire government is represented by one wheel. What purpose would three other wheels serve? This might have been a good analogy if we were talking about four separate countries, each with its own wagon wheel helping to support the world-wagon—but that’s a different analogy. Choice (B) basically restates the analogy—and is the correct answer. Choices (C) and (D) extend the analogy in some meaningless way.

22. D

The map of the United States shows which areas have a high number of non-English-speaking children. You’ll notice that the highest concentrations tend to be on the outside edges of the country, not the interior. Question 22 asks you to pick the best geographic explanation for the areas that have the highest numbers of non-English-speaking children. Choice (B) is irrelevant because we aren’t concerned with the total number of children in the country—only the children who can’t speak English. Choice (C) is irrelevant because, according to the map, some of the heaviest areas of non-English-speaking kids are in the north, such as New York and Washington. Choice (A) seems possible at first, because lower income taxes might attract new immigrants, but then again, wouldn’t they attract just about everyone? Let’s hold onto this as we look at (D): If recent immigrants settled in border and coastal states, wouldn’t that be a pretty logical explanation for why these areas contain the most non-English-speaking kids? Choice (D) is a much better answer than (A).

23. B

All of the answer choices would have an effect on the distribution of these kids EXCEPT (B), climate. In general, new immigrants would seem to make decisions based more on the availability of ESL programs, immigration laws, social services, and even migration patterns (where immigrants have gone before) than on climate. One of the states with the largest number of non-English speakers (according to the chart) is New York (which has a colder climate) while others include California and Florida, with warmer climates.

24. D

To answer this question, you have to consider each answer choice in turn. Are non-English-speaking children evenly distributed across the entire country? The map says no. Eliminate (A). Are there more of these kids in Arizona than in California? No, according to the map, California has the highest level of counties with at least 500 non-English-speaking kids. Arizona has between five and nine counties with 500 or more non-English-speaking children, but not as many as California, so we can eliminate (B). And while Texas and California do have the highest levels of kids who don’t speak English, could that really mean that they have more kids who can’t speak English than kids who can? The map does not show information that compares these two groups of children, so eliminate (C). The correct answer is (D) because it best reflects what the map tells us: In the middle states in the north of the United States, there are fewer non-English-speaking children.

25. B

The passage tells us that the Constitution and other laws “made in accordance” with the Constitution can’t be set aside by judges or any other laws. So, which is an example of this? The correct answer is (B), which says that the Bill of Rights (the first ten amendments of the Constitution) can’t be denied by any arm of the government. Choices (A), (C), and (D) are all irrelevant to this concept.

26. A

Lowering the speed limit, limiting senators to two terms, reducing pollution, or creating town zoning—none of these deprives people of their constitutional rights. However, prohibiting protests by a union takes away a fundamental right guaranteed by the Constitution: the right to free speech. Thus, the correct answer is (A).

27. B

Freedom of religion is a fundamental right guaranteed by the Constitution—so we can eliminate (A). A right to a trial by jury is also guaranteed by the Constitution—so we can eliminate (C). The right to vote is also guaranteed to citizens by the Constitution—so we can probably eliminate (D), because even if they don’t speak English, people can be citizens. The correct answer is (B).

28. D

The passage states that the economics professors say there are too many factors to be able to make long-term stock-market predictions. Choice (D) accurately restates that idea. Choices (A), (B), and (C) all make statements that contradict the passage.

29. C

Because so many different factors can affect stock prices, there can be no guarantee that two companies producing the same goods will do equally well—so we can eliminate (A). While amateur investors are mentioned in the passage, their behavior is not the most important factor on a stock’s success—so eliminate (B). The passage says that amateurs may do as well as a professional stock trader, but it does not say that they will do better—so we can eliminate (D). The correct answer must be (C), because past success does not ensure future performance—a concept always mentioned in stock prospectuses, but often forgotten in practice.

30.

Since laissez-faire economics is free from government restrictions, labor laws, building codes, and antitrust laws would all be incorrect choices. Deregulation (a), free trade (d), and non-interference (f) all imply a lack of government oversight in business.

31. C

What caused the end of the era of transatlantic ship-crossings? Do you think it was a series of accidents, a major war that disrupted routes, the cost of travel by boat, or the fact that sea travel fell out of fashion? Nope, it was something much more basic—the airplane. The correct answer is (C).

32. A

In this cartoon, the two Civil War veterans, one African American, one white—each missing a leg—are shaking hands. Always read the caption carefully in a cartoon or photograph. The two soldiers are calling each other comrade and saying they each lost a leg for a good cause. The best answer is (A), which reflects what the Civil War was fought over: the right of all men to be free and equal to each other. Both (C) and (D) suggest information that is impossible to know about these two individuals, while not reflecting the point of the cartoon. Choice (B), while true, again did not reflect the larger point behind the conflict.

33. B

The flowchart indicates that Congress is responsible for passing laws. They do not veto; that is the president’s role. TAB 2 indicates that the Judicial Branch gained power in Marbury vs. Madison, so eliminate (C). Choice (D) is tempting, but there is no indication that the Congress nominates judges. The president does this.

34. D

In TAB 3, Jefferson states that “judges as the ultimate arbiters of all constitutional questions” is “a very dangerous doctrine.” He made this statement in response to Marbury vs. Madison. The quote also indicates that “their power is…dangerous.” This matches best with (D). Choices (A) and (C) are too positive in tone to match with the Jefferson quote. Choice (B) is close, but Marbury vs. Madison pertains to the Judicial Branch, not the Executive Branch.

35. A

Jefferson is using negative language, such as “dangerous.” The only negative word in these answers is “tyranny.” It is not a literal definition of “oligarchy,” but is the only word that captures the spirit of Jefferson’s opinion regarding Marbury vs. Madison.

SCIENCE

1. D

The question states that hurricanes gather strength over warm water, so a hurricane should gather the most strength over the warmest body of water. Since the Gulf of Mexico has the highest average temperature of the bodies of water listed, the correct answer is (D).

2. A

According to the passage, patients must receive tests for blood type before certain kinds of transfusions because of the ways in which different red blood cells interact with each other. Therefore, blood type tests must be necessary only when there is going to be an exchange of red blood cells. Since the passage states that whole blood transfusions involve both red blood cells and plasma, blood type tests must be necessary before whole blood transfusions. Since plasma transfusions do not involve red blood cells, you can eliminate (B) and (C). The passage states that patients may receive transfusions when they have lost a lot of blood, not when they have not lost any blood. Therefore, (D) is not supported by the passage and is incorrect. Thus, (A) is the best answer.

3. 85%

According to the information given, coal turns into anthracite only after the coal has been subjected to heat and pressure for more than a million years. Thus, this type of coal is likely very dense, or hard. Additionally, the passage indicates that anthracite burns only when subjected to very high temperatures. Thus, the question most likely describes anthracite, which is 85–95% carbon, so the lowest level of carbon the coal is likely to contain is 85%.

4. D

According to the question, steel is formed when iron combines with carbon at very high temperatures. Since you need carbon to form steel, you likely do not want all of the carbon to burn up before it combines with the iron. Additionally, the form of coal that contains the highest percentage of carbon is likely to be the most effective for this process. Since anthracite burns only at very high temperatures, and contains the highest percentage of carbon of the coals listed, the correct answer is (D).

5. Peat (a); Anthracite (d). Peat turns into lignite, which in turn becomes subbituminous coal, which in turn becomes anthracite. Notice that these changes require both heat and pressure, which would result from the coal being farther and farther underground. Additionally, anthracite is millions of years old, and therefore is likely to become covered by dirt over the centuries. Therefore, peat is likely to be closest to the surface. According to the chart, the carbon with the highest level of carbon is anthracite.

6. C

According to the passage, seeds need moisture and oxygen. Additionally, while most seeds require temperatures between 15 and 27 degrees centigrade, corn seeds require even higher temperatures. There-fore, the correct answer must involve temperatures higher than 27 degrees centigrade. Eliminate (A). Since “arid” means “without moisture,” corn seeds likely will not germinate in an arid environment, so (B) is not the credited answer. Choice (C) mentions an environment that is moist, has plenty of oxygen, and is warmer than 27 degrees centigrade, so (C) is the correct answer. Since the passage states that maple trees can germinate in temperatures lower than 15 to 27 degrees centigrade, an environment that is suitable for maple trees will not necessarily be suitable for corn seeds. Thus, (D) is incorrect.

7. C

According to the passage, maple seeds can germinate in low temperatures. However, that does not necessarily mean that they can germinate in any temperature; there may still be temperatures that are either too high or too low for them to germinate. Thus, (A) is incorrect. The passage does state that corn seeds require temperatures higher than 27 degrees centigrade in order to grow, but that does not necessarily mean that corn is difficult to grow. Therefore, (B) is incorrect. The passage states that “most seeds will germinate when they have moisture.” So, some water is necessary for the germination process. Choice (C) is supported by the passage and is the credited answer. The passage states that maple seeds and corn seeds germinate in different temperatures, but does not mention the respective amounts of oxygen that they require. Choice (D) is not supported by the passage and is therefore incorrect.

8. C

The passage states that the body produces lactic acid during periods of intense activity. Therefore, you’ll want to examine the answer choices to find one that describes intense activity. Walking to work is not likely to involve intense activity, so you can eliminate (A). While (B) does describe the chess game as “intense,” this intensity is of a mental, rather than a physical nature. Since the passage is discussing physical activity, you can eliminate (B). An aerobics class is likely to involve intense physical activity, so (C) is the correct answer. A scary movie may be intense, but watching a movie does not involve physical activity, so (D) is not the credited answer.

9. A

The passage states that plants lie dormant during periods that are unfavorable for growth. Winter is generally the season least favorable for growth, so the correct answer is (A).

10. Unless volcanic activity has occurred, the oldest rock layers are the ones located deepest within the ground. Therefore, rock layer #4 is the oldest rock layer, and rock layer #1 is the newest rock layer.

11. C

According to the information in the food chain, bass do eat minnows. However, the food chain does not provide any information as to whether any other fish also eat minnows. Since minnows may have more than one predator, (A) is not supported by the information in the food chain. The food chain begins with microscopic algae and ends with the much larger bass. In general, the living things on the food chain seem to be ordered from smallest to largest. Since bass are higher on the food chain than are minnows, bass are likely larger than minnows, rather than the other way around. Therefore, you can eliminate (B). Bass eat minnows, which in turn eat copepods. Therefore, if there were no copepods, minnows would have fewer food options and would likely suffer. In turn, bass would suffer since they would have fewer minnows to consume. Choice (C) is supported by the information in the food chain and is the credited answer. Since bass may derive benefits from minnows that they could not derive from algae, there is no evidence that (D) is true.

12. Answers may vary but should include a discussion of a specific animal that uses passive protection. For example, the chameleon changes color to blend in with its surroundings. The passage states that one type of passive protection involves changing color to mimic the surrounding environment. Thus, this example agrees with the passage’s definition of passive protection. When a chameleon changes color in this manner, it is no longer easily visible to predators, and thus may escape predation.

13. C

According to the information in the question, 1 kilogram/liter = 1,000 kilograms/meter3. Divide both sides by 1,000 to find that 0.001 kilograms/liter = 1 kilogram/meter3. Now multiply both sides by 1,029 to find that 1.029 kilograms/liter = 1,029 kilograms/meter3. Therefore, the correct answer is (C).

14. Answers may vary, but a thorough explanation should discuss the fact that the researchers did not properly use the scientific method. Because they not only decreased greenhouse gas emissions, but they also took steps to ensure that citizens received better respiratory care, the resulting improvement in the citizens’ health could have been due to the respiratory care education campaign and the free clinics. In order to improve their study, the researchers would have had to change only one variable at a time, rather than both.

15. Mode, 0.43. The mode is defined as the most commonly occurring number, so in this case the researchers would need to find the mode of the weights. Since the weight 0.43 kg occurs most frequently in the list, the correct answer to the second question is 0.43.

16. 0.93

To find the mean of a list of numbers, add up all of the numbers and then divide by the number of numbers that you have. 0.2 + 0.4 + 0.2 + 0.8 + 0.9 + 1.7 + 1.3 + 1.8 + 0.9 + 1.1 = 9.3. You have ten numbers, so divide this result by ten: 9.3/10 = 0.93.

17. D

Decreasing the volume of the gas will make the denominator of the fraction smaller, and will contribute toward making the fraction as a whole larger. However, decreasing the temperature of the gas will make the numerator smaller, and will contribute toward making the fraction as a whole larger. Therefore, decreasing both the volume and the temperature of the gas will not necessarily increase the pressure within the gas, and you can eliminate (A). Increasing the volume of the gas will make the denominator of the fraction larger, and will therefore make the fraction as a whole smaller, so you can eliminate both (B) and (C). Decreasing the volume of the gas will make the denominator smaller, and will thus make the fraction as a whole larger. Increasing the temperature of the gas will make the numerator of the fraction larger, and will thus make the fraction as a whole larger. Thus, only (D) will cause an increase in the pressure within the gas.

18. A

According to the table, a combination occurs when A + B —> C. Note that this matches the reaction described in the equation. In this case, H2 = A, F2 = B, and 2HF = C. All of the other types of reactions have additions on the right sides of the equations, so (B), (C), and (D) are incorrect. The credited answer is (A).

19. B

Use the chart and estimate. According to the chart, the number of AIDS deaths in 2011 was approximately 1,800, and will be approximately 2,500 in 2020. Therefore, the increase in AIDS deaths in Africa between the two years will be about 2,500 – 1,800 = 700. The number of AIDS deaths in 2011 in South/Southeast Asia was approximately 500, and will be approximately 1,500 in 2020. Therefore, the increase in AIDS deaths in South/Southeast Asia between the two years will be about 1,500 – 500 = 1,000. Eliminate (A), since (B) gives you a larger answer. The number of AIDS deaths in 2011 in Eastern Europe was approximately 0, and will be approximately 250 in 2020. Therefore, the increase in AIDS deaths in Easter Europe between the two years will be about 250. Eliminate (C). Finally, the number of AIDS deaths in North America/Northern Europe was approximately 0 in 2011, and will be just slightly more than 0 in 2020, so you can eliminate (D). The correct answer is (B).

20. D

The type-A star has a temperature of 4,839 degrees Fahrenheit, so plug 4,839 into the formula in place of F. You then have K = image(4,839 – 32) + 273. Simplify inside the parentheses first to get K = image(4,807) + 273. Multiply the image and the 4,807 so that you have K = 2670.55 + 273. Finally, add to get K = 2943.55. The correct answer is (D).

21. B

Amplitude is a measure of the difference between the middle height of a wave to the top (or bottom) of a wave. If you look at the two waves, you will see that the distance from the middle height to the top of the Fault City wave is much greater than the distance from the middle height to the top of the Nicaragua wave. Therefore, the Fault City wave had a much greater amplitude. You can eliminate (A), and the correct answer is (B). The information in the caption indicates that a seismometer was used to measure the Fault City quake, so (C) is incorrect. Finally, the long-period surface waves at point R1 were larger during the Fault City quake than during the Nicaragua quake, so (D) is incorrect.

22. R1. Since you’re looking for a point that would allow you to identify signs of an earthquake before the earthquake actually occurred, you need a point before the earthquake. Points R2, R3, R4, and R5 are all located on the timeline at points after the earthquake. Only point R1 is before the earthquake, and is a point at which signs of seismic activity actually occur.

23. C

The passage mentions that ornithologists have observed that the archaeopteryx could perch on tree limbs, but does not state whether paleontologists agree with this observation. Eliminate (A). According to the passage, ornithologists believe that the archaeopteryx could not run quickly, but again the passage does not state whether paleontologists agree with this view, so (B) is not the credited answer. Paragraph 1 indicates that paleontologists believe that the archaeopteryx “could not sustain flight.” Thus (C) is supported by the passage and is the correct answer. Paragraph 2 states that ornithologists believe that the archaeopteryx may not have been able to walk quickly, but the passage does not state whether paleontologists agree with this view, so (D) is incorrect.

24. C

According to ornithologists, the “archaeopteryx was first and foremost a bird.” Of the animals listed in the answer choices, only the crow is a bird. Therefore, the correct answer is (C).

25. C

According to the information in the paragraph, work is equal to force multiplied by displacement. The paragraph also states that if you use a ramp to move an object rather than lifting the object straight up, then the amount of force required decreases, while displacement increases by exactly the same amount that the force decreased. Thus, the net product of force times displacement should not change. The correct answer is therefore (C).

26. D

Look at the chart, and read horizontally across the row for urine. Note that the columns for both albumin and glucose have dashes for entries, signaling that neither is generally present in urine. Therefore, the correct answer is (D), albumin and glucose.

27. A

The passage states that the foxes prey on rabbits, so if there are fewer rabbits in the forest, the fox population will suffer because one of its food sources will be depleted. Choice (A) is therefore supported by the passage and is the correct answer. The passage indicates that the virus infects only rabbits, so you can eliminate (B). Since the foxes rely on the rabbits for food, the foxes are not in competition with the rabbits for food, so you can eliminate (C). Finally, because the virus infects only rabbits, the foxes are unlikely to evolve to become more resistant to disease under these circumstances, so (D) is not the credited answer.

28. B

According to the introductory sentence, a solar eclipse occurs when the sun cannot be seen from the earth. Choice (A) claims that a solar eclipse occurs when the moon is no longer visible from the earth, so you can eliminate (A). Based on the information in the figure, a solar eclipse occurs when the moon is situated directly between the earth and the sun. Note that in the figure, parts of the earth are in the umbra, or deepest part of the shadow, of the moon. Since this is the situation that (B) describes, (B) is the correct answer. If light rays intercepted one another, the resulting light would likely be brighter, rather than darker, so (C) is not the credited answer. The figure shows that the earth, not the sun, is in the moon’s umbra, so you can eliminate (D).

29. B

According to the passage, penicillin prevents the growth of new bacteria. Therefore, the correct answer should involve an illness caused by bacteria. The chart indicates that streptococcal bacteria cause strep throat. Therefore, penicillin is likely to be an effective treatment for strep throat.

30. D

The question asks you to find the quote that supports the idea that simply preventing those with microbial diseases from having physical contact with others will not stop the spread of such microbial diseases. Choice (A) discusses treating the diseases with penicillin, rather than preventing direct physical contact, so (A) is not the credited answer. Choice (B) also discusses the effectiveness of penicillin, rather than the effectiveness of preventing physical contact, so (B) is incorrect. Choice (C) mentions only the different types of microbes, and does not discuss the effectiveness of preventing physical contact, so you can eliminate (C). Choice (D) mentions several ways that microbes can invade hosts: through the air, through direct or indirect physical contact, through blood, and through other bodily fluids.

31. D

According to the passage, both plants were green and were hybrids, so you can eliminate answers (A), (B), and (C), leaving only (D). If both parents carry a hidden yellow gene, then some of the offspring may be yellow, so the passage supports (D).

32. A

The passage states that in hybrids the dominant gene hides the expression of the recessive gene. Therefore, an individual who has both the dominant gene for dimples and the recessive gene for a lack of dimples will have dimples. The correct answer is (A).

33. C

According to the graph, the temperature increases, then remains constant while the sample melts, then increases, then stays constant while the sample boils, and then increases. Since this is the scenario that (C) describes, the correct answer is (C). Note that a horizontal line means that the variable on the y-axis, in this case, temperature, is not changing.

34. Point C represents the sample at the point when it is between melting and boiling. Such a sample would be liquid, so if you selected a point near point C, then you selected the correct region of the graph. Before the sample melted, it would be entirely solid, so the region of the graph near point A would not represent a time during which the sample would be liquid. During melting, some of the sample would be solid and some of the sample would be liquid, so the region of the graph near point B would not represent a time during which the sample would be liquid. During boiling, some of the sample would be liquid and some of the sample would be gas, so the region of the graph near point D would not represent a time during which the sample would be liquid. After boiling, the sample would be entirely gas, so the region of the graph near point E would also not represent a time during which the sample would be liquid.

35. B

The passage states that bees are important because they pollinate flowering plants, so (B) is supported by the passage. While (A) describes another role of bees, that role is not mentioned in the passage. Choices (C) and (D) do not describe actions that bees take, and are therefore both incorrect.